Rand des Bildes ist Teilmenge des Bildes des Rands

Neue Frage »

Zellerli Auf diesen Beitrag antworten »
Rand des Bildes ist Teilmenge des Bildes des Rands
Es seien und Gebiete in mit (denke ), sei beschränkt und es sei eine in holomorphe Funktion.

Jetzt soll ich zeigen:

Ich habe keinen Schlachtplan. Ich habe nichtmal Sätze darüber finden können, wie welche Werte auf dem Rand angenommen werden.

Was können holomorphe Funktionen noch, wovon ich nichts weiß?

Der nächste Teil ist ein Beispiel zu finden, das zeigt, dass im Allgemeinen keine Gleichheit gilt. Aber nichtmal das gelingt mir, egal wie ich das Gebiet ansetze (kreisrund, Flickenteppich oder quadratisch) und welche Funktion ich nehme (von einfach bis gebrochen über trigonometrisch und e).


Bin für jeden Schuppser dankbar. Das schlaucht mich, dass ich da so lange drüber grüble und blättere... Augenzwinkern
gonnabphd Auf diesen Beitrag antworten »

Hi,

Ich würde unterscheiden zwischen f konstant und f nichtkonstant und einen Widerspruchsbeweis bei zweiterem versuchen:

Zeige, dass (ich hoffe, hier täusche ich mich nicht...)
Dann nimm an, dass es ein gibt und führe das zum Widerspruch indem du benutzt, dass nichtkonstante holomorphe Funktionen offen sind.

Für ein Gegenbeispiel könntest du z.B. verwenden, dass jeder Punkt eine Wurzel besitzt mit nichtnegativem Imaginärteil.

Gruss Wink
Zellerli Auf diesen Beitrag antworten »

Danke für deine Überlegungen!

Ich weiß aber noch nicht so ganz, wo das hinführt...

Bei deiner ersten Vermutung täuschst du dich sicher nicht, denn - setzt man mal voraus, dass das gilt, was ich zeigen soll - es ist:
Allerdings würde mir auf Anhieb nicht einfallen, wie man den Teil anders zeigen kann (also ohne Verwendung des zu beweisenden Resultats Augenzwinkern ).

Mit der Widerspruchsannahme meinst du , oder?
Was bringt mir da die Offenheit (die geht schnell mit der Gebietstreue)? Das sagt mir erstmal nur, dass und Gebiete, also auch offen sind. Das bringt mir, dass .

Aber wenn nützt mir ja die Offenheit nichts, weil der Rand keine offene Menge ist.


Der Wurzelfunktion werde ich mal nachgehen. Nicht schlecht, darauf bin ich nicht gekommen...
gonnabphd Auf diesen Beitrag antworten »

Hmm, irgendwie hab ich da falsch gedacht. verwirrt

Öhm, sry, aber ich seh's grad auch nicht. Das Gegenbeispiel, welches ich im Sinn hatte, sollte zumindest passen.
Zellerli Auf diesen Beitrag antworten »

Hm also , wobei der Hauptzweig des komplexen Logarithmus ist, ist holomorph für jedes Gebiet ohne die 0.

Das heißt es wird alles in die obere Halbebene abgebildet, wenn ich das richtig verstanden habe.

Aber da fällt mir keine Konstellation von Gebieten ein, sodass das Bild des Rands überhaupt etwas anderes ausspuckt als der Rand des Bildes...

Ich denke da in die falsche Richtung glaube ich.
gonnabphd Auf diesen Beitrag antworten »

Zitat:
Ich denke da in die falsche Richtung glaube ich.


Jo, im wahrsten Sinne des Wortes. Wenn jeder Punkt eine Wurzel besitzt, bedeutet das, dass surjektiv ist. Augenzwinkern In diese Richtung geht mein Gedanke.
 
 
Zellerli Auf diesen Beitrag antworten »

Nicht schlecht.

Das ginge z.B. für und , weil dann leer wäre.

Aber hier ist G nicht beschränkt. Sobald ich es beschränke, wirds schon wieder problematisch und ich kriege keinen leeren Rand hin.
Huggy Auf diesen Beitrag antworten »

Ist denn die zu beweisende Behauptung überhaupt richtig?

Dem Vorschlag von gonnabphd folgend sei f(z) gleich dem Hauptwert von gewählt. Den Hauptwert bekommt man, in dem man in Polarkoordinaten den Winkel von z aus dem Bereich wählt. Dadurch wird die negative reelle Achse auf die positive imaginäre Achse abgebildet. f ist in C\0 holomorph.

Nun sei



also das Viertel eines Kreisrings im 3. Quadranten. Dann ist



also das Achtel eines Kreisrings im 4. Quadranten.

Zu gehört das Intervall [-2, -3] auf der negativen imaginären Achse. Das ist aber nicht in enthalten, denn das Intervall [-4, -9] auf der negativen reellen Achse wird ja auf das Intervall [2, 3] auf der positiven imaginären Achse abgebildet.
Zellerli Auf diesen Beitrag antworten »

Ich nehme mal stark an, dass die zu beweisende Behauptung richtig ist (es steht dort wie gesagt: "Zeigen Sie...").

Ich bin noch nicht so weit in Funktionentheorie, dass ich jetzt mit Sicherheit sagen könnte, dass man die Wurzelfunktion so definieren muss, aber die einzig mir bekannte holomorphe Wurzelfunktion ist , wobei der Hauptzweig des komplexen Logarithmus ist. Und die ist für keine nichtpositiven reellen Zahlen definiert.
Ist die Gesamtfunktion noch holomorph, wenn man die erstgenannte für die reellen nichtpositiven Zahlen mit fortsetzt?
sergej88 Auf diesen Beitrag antworten »

Hallo,

soweit ich mich erinner, kann man den Hauptzweig auf ganz C ohne die negative reele Achse definieren. Anschliessend lässt sich dieser auf die Achse einsitig fortsetzen. Leider ist diese Fortsetzung dann aber auf der Achse nicht mehr stetig, sondern macht genau einen Sprung um
Man male sich Bilder, verwende das Argumentprinzip oder denke an Riemansche Flächen und Überlagerungen...

Zur Aufgabe, an sich ist diese auch in der reellen Analysis gültig, jedoch muss man vorraussetzen, dass f offen ist bzw. total differenzierbar und Jakobi Matrix überall invertierbar. Wobei beim 2ten es einfacher ist noch stetige diffbarkeit zu fordern.

Es ist offen. Sei jetzt

, dh:
Dann ergibt sich direkt:



Also insgesamt: , was die Inklusion zeigt.

Man sollte wohl noch begründen wieso man den "Randoperator" aus dem Urbild "rausziehen" darf. Diese Inklusion solltest du aber hinbekommen, wenn ich nichts übersehen habe geht es ganz gut direkt mit der Epsilondefinition.

Guten Tag noch
Huggy Auf diesen Beitrag antworten »

Zitat:
Original von Zellerli
Ich bin noch nicht so weit in Funktionentheorie, dass ich jetzt mit Sicherheit sagen könnte, dass man die Wurzelfunktion so definieren muss, aber die einzig mir bekannte holomorphe Wurzelfunktion ist , wobei der Hauptzweig des komplexen Logarithmus ist. Und die ist für keine nichtpositiven reellen Zahlen definiert.

Ich bin leider auch nur sehr elementar mit Funktionentheorie vertraut. Aber weshalb sollte der Logarithmus für negative reelle Zahlen nicht definiert sein. Es ist doch



Und meiner dumpfen Erinnerung nach ist der Hauptwert von . Das wird auch von Mathematica ausgespuckt.
sergej88 Auf diesen Beitrag antworten »

So ich nochmal, habe mir die Mühe gemacht und das nochmal nachgeschaut, ich fasse das hier mal kurz zusammen:

Für heißt Logarithmus von z, wenn erfüllt ist.
Natürlich gibt es davon unendlich viele, jedoch haben alle die Darstellung

mit einem Argument arg(z) halt. Dieses ist ja nur bis auf Vielfache von bestimmt, woraus folgt, dass sich 2 Logarithmen um Vielfache von unterscheiden.
Wir können z.B.
und für negative:
definieren.

Das Problem ist jetzt, dass man nicht irgendeine Zuordnung haben möchte, sonderen eine stetige oder am liebsten eine holomorphe Zuordnung. Interessanter weise kann man jetzt leicht zeigen:

Auf einem Gebiet ist folgendes äquivalent:
1. Es gibt eine stetigen Logarithmus:
2. Es gibt eine stetige Argumentfunktion auf G.
3. hat eine Stammfunktion auf G (Log ist auto. damit holomorph)
4. Die Umlaufzahl eines Weges in G, ist im Nullpunkt stehts 0.

Für einfach zusammenhängende Gebiete ist natürlich 4. erfüllt und wir haben einen Log. Die größte einf. zus. offene Teilmenge ist die aufgeschlitzte Ebene, also von der Form
fest.
Für können wir den Hauptzweig konstruieren. Dieser Log hat den Vorteil, dass er auf der reellen Achse mit dem reellen Log übereinstimmt. Andere Logarithmen müssen dieses nicht tun.
Nähern wir uns der geraden mit dem Winkel Alpha, so können wir Grenzwerte von 2 Seiten betrachten und bekommen damit 2 Fortestzungen des Logarithmuses auf . Es macht dann aber einen Unterschied von welcher Seite wir kommen, denn diese Fortsetzung ist jetzt unstetig.

Für den Hauptzweig bekommen wir aber so zumindest die bekannten Formeln wie oben bereits angegeben.


Guten Abend noch.
gonnabphd Auf diesen Beitrag antworten »

Zitat:
Original von sergej88


Hmm, es ist doch im Allgemeinen



(ausser im Falle, dass f injektiv ist) verwirrt

Einen ähnlichen Fehler hatte ich auch gemacht.

Noch eine Frage an Zellerli:

Zitat:
Original von Zellerli
Aber hier ist G nicht beschränkt.


Sollte G denn beschränkt sein? Wenn ja, dann hätte ich vermutlich sogar eine Idee, wie man den Beweis führen könnte.

2. Frage falls G beschränkt sein muss. Findet jemand ein Gegenbeispiel für unbeschränkte Gebiete? Irgendwie habe ich das Gefühl sowas oszilierendes wie der Sinus (bzw. die e-Funktion) könnte klappen. Jedenfalls ist daran jeder Beweisversuch von mir gescheitert. (oder sogar etwas, was immer schnell zu oszilieren beginnt für |z| gegen Unendlich.

Edit: Hab' grade die Aufgabenstellung nochmal gründlich gelesen... Hammer
gonnabphd Auf diesen Beitrag antworten »

So: Hier noch mein Tipp.

Ich wüde es dann doch direkt zeigen: Sei . Dann gibt es eine Folge mit .

Betrachte nun eine Folge mit . Zeige damit, dass es ein gibt, mit . Hinweis: ist beschränkt.

Nachtrag: Ein Gegenbeispiel für unbeschränkte Gebiete findet man übrigens mit der Exponentialfunktion und G = Omega = C. Dann ist 0 im Rand des Bildes, aber der Rand von G ist leer. Man kann G auch beliebig anders wählen, solange es gegen minus Unendlich unbeschränkt ist.
Zellerli Auf diesen Beitrag antworten »

Habe jetzt gerade nicht viel Zeit das ausführlich durchzuarbeiten, aber geht nicht, weil gelten soll.
gonnabphd Auf diesen Beitrag antworten »

Für ist .
Zellerli Auf diesen Beitrag antworten »

Ja richtig. Offen und abgeschlossen. Cheater! Augenzwinkern

Weil beschänkt ist, muss es ein geben mit ?
Das muss ich mir nochmal zu Gemüte führen...


Und danke, sergej88, dass du den Log nochmal so genau beleuchtet hast.
Hier übrigens mal graphisch einsichtig, wie das mit den ungleichen Grenzwerten an der negativen reellen Achse aussieht: http://upload.wikimedia.org/wikipedia/commons/d/d8/Complex_log.jpg
Neue Frage »
Antworten »



Verwandte Themen

Die Beliebtesten »
Die Größten »
Die Neuesten »